2017 AMC 8 Problems/Problem 4

Revision as of 15:04, 22 November 2017 by LearningMath (talk | contribs) (Solution:)

Problem 4

When $0.000315$ is multiplied by $7,928,564$ the product is closest to which of the following?

$\textbf{(A) }210\qquad\textbf{(B) }240\qquad\textbf{(C) }2100\qquad\textbf{(D) }2400\qquad\textbf{(E) }24000$

Solution:

We can approximate $7,928,564$ to $8,000,000,$ and $0.000315$ to $0.0003.$ Multiplying the two yields $2400.$ This gives our answer to be $\text{D)}$ $2400.$